LSAT and Law School Admissions Forum

Get expert LSAT preparation and law school admissions advice from PowerScore Test Preparation.

User avatar
 Dave Killoran
PowerScore Staff
  • PowerScore Staff
  • Posts: 5852
  • Joined: Mar 25, 2011
|
#80533
This game is also discussed in our Podcast: LSAT Podcast Episode 70: The May 2020 LSAT-Flex Logic Games Section

Complete Question Explanation
(The complete setup for this game can be found here: https://forum.powerscore.com/lsat/viewtopic.php?t=33050)

The correct answer choice is (B).

The question stem asks you to replace the fourth rule with a different rule that has an identical effect, with no additional effects. The fourth rule created a not-block between G and J, and thus we must seek to replicate that result in the answers. As is often the case with Rule Substitution questions, at a glance it's difficult to imagine what route the test makers might choose, but the best place to start is to determine what you know about each variable in the rule being suspended.

In this case, we know that G cannot play in the first game due to the first rule, and J cannot play in the third game due to the second rule. Thus, as Kelsey notes below, "the only game in which G and J could possibly be together is the 2nd game. So we don't really have to worry about if J is in the 1st game or if G is in the 3rd game. To keep them apart, we just have to show that they cannot both be in the 2nd game. That's what answer choice (B) gives us: if J is in the 2nd game, G is in the 3rd tells us that J and G cannot be together in the 2nd game which means they cannot be together in any game!"


Answer choice (A): This answer does force G and J apart when H plays in the first game. However, if H doesn't play in the first game, then G and J can play together in the second game (with F and H in the third game, and L and M in the first game). Thus, since this answer does not always keep G and J apart, it is incorrect.

Answer choice (B): This is the correct answer choice. This is an unusual correct answer since it directly addresses the exact two variables in the suspended rule. That's not normally how LSAC likes to make answer choices for these questions. However, as noted above, this answer separated G and J from being together in the second game—the only game where they could appear together—and is thus correct.

Answer choice (C): This rule creates a horizontal JF block. However, when J is second then G and J can play together in the second game (with F and H in the third game, and L and M in the first game). Thus, since this answer does not always keep G and J apart, it is incorrect.

Answer choice (D): This rule creates a horizontal JG block. This clearly meets the condition of keeping G and J apart, and so it appears promising at first. However, it also creates additional effects that go beyond the original fourth rule. For example, one of G or J always plays in the second game now, which was not an original condition.

Note as well how similar this answer is to (B). Answer choice (B) creates this horizontal block relationship just when J plays in the second game, whereas this answer creates this relationship when J plays in the first and second games. That extra difference—J playing the first game—overextends this answer and makes it incorrect.

Answer choice (E): This answer is immediately suspect since Template #4 allowed G and L to play in the same game. And that problem is enough to eliminate the answer.

Additionally, this rule still allows G and J to play together in the second game (with F and H in the third game, and L and M in the first game). Thus, since this answer does not always keep G and J apart, it is incorrect for that reason as well.
 lina2020
  • Posts: 20
  • Joined: Jul 23, 2020
|
#80847
Hi PowerScore,

By process of elimination, I was able to eliminate answers C, D, and E but did not understand how to approach A and B. The issue I had with picking A or B, is that they are both conditional and only activated if the sufficient condition is met, which I did not think would serve as an effective substitute. For example, "if Jasmine plays in the second game" would only apply in that one case but what if she plays in the first game?

Also, in general I had trouble fully determining what the exact effect of the G + J not block resulted in other than applying the rule on the case by case basis, once F had been placed in the game.

Would you please list your thought process on this question and answers A and B in detail. Thank you!
User avatar
 KelseyWoods
PowerScore Staff
  • PowerScore Staff
  • Posts: 1079
  • Joined: Jun 26, 2013
|
#80880
Hi lina2020!

Remember that from the first 2 rules, G cannot be in the 1st game and J cannot be in the 3rd game. That means the only game in which G and J could possibly be together is the 2nd game. So we don't really have to worry about if J is in the 1st game or if G is in the 3rd game. To keep them apart, we just have to show that they cannot both be in the 2nd game. That's what answer choice (B) gives us: if J is in the 2nd game, G is in the 3rd tells us that J and G cannot be together in the 2nd game which means they cannot be together in any game!

Hope this helps!

Best,
Kelsey
User avatar
 TootyFrooty
  • Posts: 74
  • Joined: Oct 13, 2023
|
#104830
I'm having a bit of trouble with rule substitution questions. I've reviewed the chapter twice, and even taken notes. It makes sense with the simple examples given in the book but the moment it gets more complicated, such as in this scenario I'm like a lost raccoon.

In this example. I'm also struggling to understand how B is correct when the G/J block doesn't really restrict G from being in other two rows and doesn't restrict J from being in the other two. So my understanding of rule substitution is that it would have the same EXACT effect if it were replaced by the rule but every time I see the answer I don't really see that happening. Answer choice b restricts Gary to place 3, when the rule makes no such suggestions or blocks. Am I misunderstanding the concept?
User avatar
 Hanin Abu Amara
PowerScore Staff
  • PowerScore Staff
  • Posts: 60
  • Joined: Mar 29, 2023
|
#105484
Hi TootyFrooty,

I understand your frustrations. Rule substitution questions can be quite difficult.

You’re always thinking about the effect of the rule. In our example we’re replacing the rule that G and J can’t be together. So let’s think about what this may means.

If we were to split the game into two board we would have:

Board 1: F and G/H block on 2

Board 2: F and G/H block on 3

Now on board 1, J will never be with G regardless because G is either on 2 or 3 and J has to be on one.

J. F. G/H
L G/H. M
1. 2. 3

BUT on board 2 you have this

—- ——. F
——. ——. G/H

So to avoid the J and G rule problem we either have to put G on 3 with F OR ensure that if G is on 2 (since it can’t be on 1) J is on 1.

The structure of the rule doesn’t so much matter as much as the effect of the rule.

Hope that makes sense.

Get the most out of your LSAT Prep Plus subscription.

Analyze and track your performance with our Testing and Analytics Package.